LSAT and Law School Admissions Forum

Get expert LSAT preparation and law school admissions advice from PowerScore Test Preparation.

 falconbridge
  • Posts: 14
  • Joined: May 05, 2020
|
#75479
Hello,

Unfortunately, I'm not understanding why I got this question wrong. The questions reads: "Which one of the following statements concerning the Concert of Europe at the end of the first paragraph can most reasonably be inferred from the passage?"

The correct answer is D, which reads: "Equilibrium in the system was maintained as members grouped together to counterbalance mutual threats."

I chose E, which reads: "It was more stable than most multipolar systems because its smaller members reacted promptly to aggression by its larger members."

The portion of the passage that I believe is relevant is: "Frequent, small confrontations are one attribute of multipolar systems and are usually the result of less powerful members grouping together to counter threats from larger, more aggressive members seeking hegemony. Yet the constant and inevitable counterbalancing typical of such systems usually results in stability. The best-known example of a multipolar system is the Concert of Europe, which coincided with general peace on that continent lasting roughly 100 years beginning around 1815."

My thought process was such: (1) one attribute of multipolar systems is less powerful members grouping together to counter threats from larger, more aggressive MEMBERS seeking hegemony ---> (2) this typically results in stability ---> (3) the best-known example of a multipolar system is the Concert of Europe, so if (1) causes (2) and the Concert of Europe is one of the best examples of a multipolar system, maintaining peace for a century, then it must have been more stable the most other systems because of (1).

I understand that my long-winded explanation makes it appear as though I greatly over-thought this; I didn't, this all happened in my head in a matter of seconds, I'm just bad at writing concise explanations.

Can someone please explain to me why E is wrong?

Much thanks,

Falcon
 Paul Marsh
PowerScore Staff
  • PowerScore Staff
  • Posts: 290
  • Joined: Oct 15, 2019
|
#75709
Hi Falcon!
the Concert of Europe is one of the best examples of a multipolar system
This is an assumption - we don't know this from the passage. The passage tells us that the Concert of Europe is the best-known example of a multipolar system; that doesn't mean it was the most stable or successful multipolar system. Just because something is super well-known, that doesn't mean it's the best! (See: Ed Sheeran). Maybe most multipolar systems were actually more stable even though they're less well known. We just don't know. The passage doesn't tell us anything about how stable other multipolar systems are, so we have no reason to assume that the Concert of Europe was more stable than them.

When we're making inferences from the passage, we want to point to the part of the passage that specifically suggests that inference without making assumptions. Since (E) would require us to assume information not supported by the passage, it's incorrect.

Hope that helps!

Get the most out of your LSAT Prep Plus subscription.

Analyze and track your performance with our Testing and Analytics Package.